Đến nội dung

Hình ảnh

VMF's Marathon Hình học Olympic

* * * * * 1 Bình chọn hình học

Lời giải halloffame, 02-01-2018 - 16:29

Lời giải bài toán 196. Ta chứng minh bài toán cho đường tròn $(K)$ tiếp xúc trong $(O),$ trường hợp tiếp xúc ngoài chứng minh tương tự. Ta thấy có thể bỏ đi điểm $B$ không cần thiết.

Bài toán 196'. $\Delta ADC$ vuông tại $D$ nội tiếp $(O),$ một đường tròn $(E)$ tiếp xúc trong $(O)$ ở $T.M,N \in (E)$ sao cho $MN \parallel AD$ và $MN=AD.P,R$ là trung điểm $MD,MC.$ Khi đó $P \in (ORT).$

Chứng minh. 

$M'$ đối xứng $M$ qua $T.$ Dựng điểm $I$ sao cho $OEMI$ là hình bình hành.

$OI$ cắt $(O),CD$ ở $K,L.J$ là hình chiếu $I$ lên $CD.$

Từ $OEMI$ là hình bình hành và $EM=ET$ ta suy ra được $IK=IM=JN,LK=LM$

Gọi $Q$ đối xứng $M$ qua $O$ thì $Q \in LM'.$ Ta có $LM'.LQ=LK.NJ=LK.KI=KO^2-OL^2=LC.LD \Rightarrow Q \in (M'CD).$

Qua phép vị tự tâm $M$ tỉ số $\frac{1}{2}$ ta có ngay đpcm.

[attachment=33194:Screen Shot 2018-01-02 at 1.29.42 AM.png]

Đi đến bài viết »


  • Please log in to reply
Chủ đề này có 434 trả lời

#21
mrjackass

mrjackass

    Trung sĩ

  • Thành viên
  • 110 Bài viết

Lời giải Bài 9

3vJIT1A.png

Giả sử $\Delta ABC$ nằm trên mặt phẳng phức với gốc tọa độ là $O$ và $(ABC)$ là đường tròn đơn vị. Kí hiệu các điểm bởi chữ cái viết thường của nó.

Khi đó:$o=0$, trọng tâm $\Delta ABC$ là $\frac{a+b+c}{3}$ suy ra $h=a+b+c$.

Do $A,B,C$ nằm trên đường tròn đơn vị nên $|a|=|b|=|c|=1$ suy ra $a=\frac{1}{\bar{a}}$, $b=\frac{1}{\bar{b}}$ và $c=\frac{1}{\bar{c}}$.

Phương trình đường thẳng $AB$ là $\begin{vmatrix} a & \bar{a} & 1\\ b & \bar{b} & 1\\ p & \bar{p} &1 \end{vmatrix}$ hay $p+ab\bar{p}=a+b$. Ta có phương trình tương tự cho $AC$ là $q+ac\bar{q}=a+c$

Vì $M$ cùng phía $BC$ nên $\Delta MBC$ đồng dạng ngược hướng $\Delta HPQ$. từ đó ta có đẳng thức $\frac{m-b}{b-c}=\frac{\bar{h}-\bar{p}}{\bar{p}-\bar{q}}$ suy ra $m=(b-c)\left ( \frac{\bar{h}-\bar{p}}{\bar{p}-\bar{q}} \right )+b$

Việc chứng minh $MH\perp PQ$ tương đương với chứng minh $\frac{m-h}{p-q}$ là số thuần ảo hay $\frac{m-h}{p-q}=\frac{\bar{h}-\bar{m}}{\bar{p}-\bar{q}}$

Từ đẳng thức $m=(b-c)\left ( \frac{\bar{h}-\bar{p}}{\bar{p}-\bar{q}} \right )+b$ suy ra 

$$(m-h)(\bar{p}-\bar{q})=(b-c)(\bar{h}-\bar{p})+(b-h)(\bar{p}-\bar{q})=(b-c)(\bar{h}-\bar{p})-(a+c)(\bar{p}-\bar{q})$$

$$(\bar{h}-\bar{m})(p-q)=(\bar{h}-\bar{b})(p-q)-(\bar{b}-\bar{c})(h-p)=(\bar{a}+\bar{c})(p-q)-(\bar{b}-\bar{c})(h-p)$$

Ta có 

$$(b-c)\left ( \bar{h}-\bar{p} \right )-(a+c)(\bar{p}-\bar{q})=(b-c)\left ( \frac{1}{a}+\frac{1}{b}+\frac{1}{c}-\frac{a+b-p}{ab} \right )-(a+c)\left ( \frac{a+b-p}{ab}-\frac{a+c-q}{ac} \right )=(b-c)\left ( \frac{1}{c}+\frac{p}{ab} \right )-(a+c)\left ( \frac{ac-ab+qb-pc}{abc} \right )=\frac{(b-c)(ab+pc)-(a+c)(ac-ab+qb-pc)}{abc}=\frac{a^2b-a^2c+ab^2-abq-ac^2+acp+bcp-bcq}{abc}$$

$$(\bar{a}+\bar{c})(p-q)-(\bar{b}-\bar{c})(h-p)=\left ( \frac{1}{a}+\frac{1}{c} \right )(p-q)-\left ( \frac{1}{b}-\frac{1}{c} \right )(a+b+c-p)=\frac{(a+c)(p-q)b+a(b-c)(a+b+c-p)}{abc}=\frac{a^2b-a^2c+ab^2-abq-ac^2+acp+bcp-bcq}{abc}$$

Vậy từ 2 điều trên suy ra 

$$(b-c)(\bar{h}-\bar{p})-(a+c)(\bar{p}-\bar{q})=(\bar{a}+\bar{c})(p-q)-(\bar{b}-\bar{c})(h-p)$$

hay $\frac{m-h}{p-q}=\frac{\bar{h}-\bar{m}}{\bar{p}-\bar{q}}$ hay ta có điều cần chứng minh

 

Đề xuất Bài 10 (AoPS): Chứng minh rằng trong một tam giác nếu lấy đổi xứng của một tiếp tuyến của đường tròn nội tiếp qua các phân giác ngoài thì các đường thẳng đối xứng này cắt nhau tạo thành một tam giác bằng tam giác đã cho . 

$$\begin{array}{| l | l |} \hline Ngockhanh99k48 & 1\\ \hline IHateMath & 1\\ \hline fatcat12345 & 2\\ \hline dogsteven & 2\\ \hline baopbc & 3\\ \hline QuangDuong12011998 & 1\\ \hline xuantrandong & 1\\ \hline mrjackass & 1\\ \hline\end{array}$$


Bài viết đã được chỉnh sửa nội dung bởi baopbc: 28-05-2016 - 09:17

420 Blaze It Faggot


#22
quanghung86

quanghung86

    Thiếu úy

  • Điều hành viên
  • 632 Bài viết

Nguồn gốc bài 9 http://www.artofprob...unity/c6h486466

Nguồn gốc bài 10 http://www.artofprob...unity/c6h470341



#23
mrjackass

mrjackass

    Trung sĩ

  • Thành viên
  • 110 Bài viết

Vì thầy đã cho link nên em xin đề xuất lại 

Đề xuất $\boxed{\text{Bài 10'}}$ Cho $\Delta A_0B_0C_0$ nhọn trọng tâm $G_0$. $A_0G_0,B_0G_0,C_0G_0$ cắt lại đường tròn $(A_0B_0C_0)$ tại $A_1,B_1,C_1$. Trọng tâm $A_1B_1C_1$ là $G_1$. Tiếp tục làm tương tự như trên để có tam $A_2B_2C_2$, $A_3B_3C_3$,...

Chứng minh rằng $lim_{n\rightarrow \infty }\widehat{A_nB_nC_n}=lim_{n\rightarrow \infty }\widehat{B_nC_nA_n}=lim_{n\rightarrow \infty }\widehat{C_nA_nB_n}=60^{\circ}$

Nguồn : Sáng tác 


Bài viết đã được chỉnh sửa nội dung bởi bangbang1412: 27-05-2016 - 23:32

420 Blaze It Faggot


#24
quanghung86

quanghung86

    Thiếu úy

  • Điều hành viên
  • 632 Bài viết
Bài toán 10' tương đương với việc cần chứng minh $\lim_{n\to\infty}OG_n=0$ với $G_n$ là trọng tâm tam giác $A_nB_nC_n$ và $O$ là tâm đường tròn ngoại tiếp tam giác $A_0B_0C_0$.
 
Bài toán này lần đầu tiên xuất hiện ở đây https://cms.math.ca/...x_v9n05_May.pdf bài toán 844 trang 143
 
Lời giải có ở đây https://cms.math.ca/..._v10n08_Oct.pdf trang 264
 
Chú ý bài toán cho không gian được giải bởi thầy Nguyễn Minh Hà với kỹ thuật đó cũng giải được cho phẳng, bài báo của thầy Hà ở đây https://cms.math.ca/crux/v36/n8/
 
Bài toán chúng ta đóng góp ở đây không nhất thiết mới, đưa lên để thảo luận, tìm lời giải mới. Nếu nguồn gốc ở đâu thì ghi rõ ở đó, còn nếu không nhớ hoàn toàn có thể ghi "sưu tầm", ai nhớ nguồn gốc thì dẫn lại là chuyện bình thường, không có vấn đề gì cả.
 
$\boxed{\text{Bài toán 11.}}$ Cho tam giác $ABC$ nội tiếp đường tròn $(O)$ và $P$ di chuyển trên cung $BC$ không chứa $A$. Đối xứng của $PA$ qua $PB,PC$ lần lượt cắt $AB,AC$ tại $F,E$. Một đường thẳng vuông góc với $PA$ tại một điểm chia $PA$ tỷ số cố định cắt tiếp tuyến tại $A$ của $(AEF)$ tại $Q$. Chứng minh rằng $Q$ luôn thuộc đường thẳng cố định khi $P$ thay đổi.

Nguồn gốc: Sáng tác.

Bài viết đã được chỉnh sửa nội dung bởi quanghung86: 27-05-2016 - 23:11
Sửa đề!


#25
mrjackass

mrjackass

    Trung sĩ

  • Thành viên
  • 110 Bài viết

Dạ em muốn đính chính là em không lấy bài từ đâu, nghịch GSP và nhìn ra thôi ạ.


Bài viết đã được chỉnh sửa nội dung bởi baopbc: 30-05-2016 - 12:08

420 Blaze It Faggot


#26
quanghung86

quanghung86

    Thiếu úy

  • Điều hành viên
  • 632 Bài viết

Cũng không vấn đề gì cả, mình sau này có thể tự nghĩ ra những bài đã có từ trước là chuyện rất bình thường, thầy đã bị thế nhiều. Bởi vậy việc đưa lên thảo luận là quan trọng vì nếu ai biết thì trích dẫn lại rõ ràng bài này đầu tiên có ở đâu để mình biết :)!



#27
baopbc

baopbc

    Himura Kenshin

  • Thành viên nổi bật 2016
  • 410 Bài viết

$\boxed{\text{Lời giải bài 11}}$ Gọi $A'$ là giao của $AO$ với $BC.K$ là giao của $BC$ với $EF$.

Dễ thấy $PB,PC$ lần lượt là phân giác $\angle APF,\angle APE$ nên theo định lí $Menelaus$ ta suy ra $\frac{KE}{KF}=\frac{PE}{PF}\Longrightarrow PK$ là phân giác ngoài của $\angle EPF$.

Post 168.png

$\Longrightarrow \angle APK=\angle APE-\angle EPK=2\angle APC-90^\circ+\angle EPF/2=2\angle ABC-90^\circ+\angle BAC$

$=\angle B+90^\circ-\angle ACB=\angle AA'K \Longrightarrow A,A',P,K$ đồng viên.

$\Longrightarrow APA'=\angle AKA'$. Mặt khác biến đổi tỉ số dễ suy ra $KB$ là phân giác $AKF$ nên $\angle APA'=(BC,EF)$.

Gọi $I$ là tâm $\odot (AEF)$ thì $(AI,AO)=(BC,EF)=\angle APA'.M,N$ lần lượt là hình chiếu của $Q$ lên $AP,AO$.

Dễ thấy $A,N,M,Q$ đồng viên nên $\angle AMN=\angle AQN=90^\circ-\angle A'AQ=(AI,AO)=\angle APA'\Longrightarrow MN\parallel A'P$.

Mặt khác $\frac{AM}{MP}$ không đổi nên $\frac{AN}{NA'}$ không đổi $\Longrightarrow N$ cố định hay $Q$ thuộc một đường thẳng cố định. $\blacksquare$

Bài toán đề nghị.

$\boxed{\text{Bài toán 12}}$. Cho tam giác $ABC$. Một đường tròn bất kì qua $B,C$ cắt $AC,AB$ tại $E,F.EF$ cắt $BC$ tại $L.P$ là một điểm trên $BC$. Lần lượt lấy $M$ thuộc $AP,H$ thuộc $EF$ sao cho $L,H,M,P$ đồng viên. $\odot (MEF)$ cắt $AP$ tại $T$, Kẻ $TI\parallel BC$ sao cho $I$ thuộc $\odot (MEF)$. Chứng minh rằng $\angle HAF=\angle IAE$.

Nguồn.

$$\begin{array}{| l | l |} \hline Ngockhanh99k48 & 1\\ \hline IHateMath & 1\\ \hline fatcat12345 & 2\\ \hline dogsteven & 2\\ \hline baopbc & 4\\ \hline QuangDuong12011998 & 1\\ \hline xuantrandong & 1\\ \hline mrjackass & 1\\ \hline\end{array}$$


Bài viết đã được chỉnh sửa nội dung bởi baopbc: 28-05-2016 - 09:18


#28
dogsteven

dogsteven

    Đại úy

  • Thành viên
  • 1567 Bài viết

$\boxed{\text{Lời giải bài toán 12}}$

Đường thẳng qua $A$ song song với $BC$ cắt $EF$ ở $Q$. $MQ$ cắt $(MEF)$ lần thứ $2$ tại $R$

Dễ thấy $\Delta QAE\sim \Delta QFA$ nên $OA^2=QE.QF$ mà $QE.QF=QM.QR$ nên $\Delta QAM \sim \Delta QRA$

Từ đó ta có $\widehat{QRA}=\widehat{QAM}=\widehat{MTI}=\widehat{QRI}$ nên $A,I,R$ thẳng hàng.

Dễ thấy tứ giác $AHMQ$ nội tiếp nên $\widehat{AHQ}=\widehat{AMQ}=\widehat{QAR}=\widehat{IAR}=\widehat{AIT}$

Từ đây dễ dàng suy ra điều phải chứng minh.

 

$\boxed{\text{Bài toán 13}}$. Cho tam giác $ABC$ ngoại tiếp $(I)$ với $D, E, F$ là các tiếp điểm của $(I)$ trên $BC, CA, AB$. $X, Y, Z$ lần lược là các chân đường phân giác xuất phát từ đỉnh $A, B, C$ của tam giác $ABC$. Đường thẳng qua $D$ vuông góc với $YZ$ cắt $AI$ tại $M$. Đường thẳng qua $E$ vuông góc với $ZX$ cắt $BI$ tại $N$. Đường thẳng qua $F$ vuông góc với $XY$ cắt $CI$ tại $P$. Chứng minh rằng $\Delta MNP = \Delta DEF$

Nguồn: Thầy Trần Quang Hùng.

$$\begin{array}{| l | l |} \hline Ngockhanh99k48 & 1\\ \hline IHateMath & 1\\ \hline fatcat12345 & 2\\ \hline dogsteven & 3\\ \hline baopbc & 4\\ \hline QuangDuong12011998 & 1\\ \hline xuantrandong & 1\\ \hline mrjackass & 1\\ \hline\end{array}$$


Bài viết đã được chỉnh sửa nội dung bởi halloffame: 30-12-2017 - 13:19

Quyết tâm off dài dài cày hình, số, tổ, rời rạc.


#29
viet nam in my heart

viet nam in my heart

    Thượng sĩ

  • Điều hành viên OLYMPIC
  • 242 Bài viết

Lời giải $\boxed{\text{Bài toán 13}}$

Bổ đề 1: Gọi $I_a,I_b,I_c$ lần lượt là tâm đường tròn bàng tiếp góc $A,B,C$ của tam giác $ABC$. Khi đó $OI_a \perp YZ$ (Đây là bổ đề quen thuộc nên em/ mình không chứng minh lại ở đây)

Bổ đề 2: $OI$ chia đôi $MD$

Chứng minh:

Gọi $X$ là điểm đối xứng của $I_a$ qua $O$. Gọi $O'$ là tâm $(I_{a}I_{b}I{c})$

Xét trong tam giác $I_a,I_b,I_c$ có $O$ là tâm đường tròn $Euler$, $I$ là trực tâm. $O'$ là tâm đường tròn ngoại tiếp

Do đó theo tính chất về tâm đường tròn $Euler$ thì $ O$ là trung điểm của $O'I$

Từ đó suy ra $O'I_{a}IX$ là hình bình hành. Suy ra $IX \parallel O'I_{a}$

Mà dễ thấy $O'I_{a} \perp BC$ suy ra $IX \parallel BC$. Do đó suy ra $I,X,D$ thẳng hàng. Khi đó theo định lý $Thales$ suy ra $OI$ chia đôi $MD$

Bổ đề 3: $DM,EN,FP$ đồng quy tại trung điểm mỗi đường

Chứng minh:

Gọi $OI$ cắt $MD$ tại $S$. Khi đó $\dfrac{SI}{OI}=\dfrac{DI}{IX}=\dfrac{DI}{I_{a}O'}=\dfrac{r}{r'}$

Trong đó $r$ là bán kính của $(I)$ và $r'$ là bán kính của $(I_{a}I_{b}I{c})$

Chứng minh tương tự suy ra $S$ cũng là trung điểm của $EN$,$FP$

anh-dep-du-lich-vinh-yen-vinh-phuc-5-1024x707jpg_Page1.jpg

Trở lại bài toán:

Áp dụng trực tiếp bổ đề $3$ suy ra $MP=DF,MN=DE,NP=EF$ suy ra điều phải chứng minh

$\boxed{\text{Bài toán 14}}$(Sưu tầm) Cho tam giác $ABC$ ngoại tiếp $(I)$. Đường tròn $(O_a)$ tiếp xúc ngoài với $(I)$ và tiếp xúc với các tia đối của tia $BA,CA$ lần lượt tại $A_1,A_2$. Gọi $d_a$ là đường thẳng đi qua $A_1,A_2$. Tương tự ta có các đường thẳng $d_b,d_c$. Giả sử các đường thẳng $d_a,d_b,d_c$ cắt nhau tạo thành tam giác $XYZ$. Gọi $M,N$ lần lượt là tâm đường tròn ngoại tiếp, nội tiếp tam giác $XYZ$. Chứng minh rằng: $M,N,I$ thẳng hàng

$$\begin{array}{| l | l |} \hline Ngockhanh99k48 & 1\\ \hline IHateMath & 1\\ \hline fatcat12345 & 2\\ \hline dogsteven & 3\\ \hline baopbc & 4\\ \hline QuangDuong12011998 & 1\\ \hline xuantrandong & 1\\ \hline mrjackass & 1\\ \hline vietnaminmyheart & 1\\ \hline\end{array}$$


Bài viết đã được chỉnh sửa nội dung bởi viet nam in my heart: 28-05-2016 - 18:31

"Nếu bạn hỏi một người giỏi trượt băng làm sao để thành công, anh ta sẽ nói với bạn: ngã, đứng dậy là thành công." Isaac Newton

VMF's Marathon Hình học Olympic


#30
Bui Ba Anh

Bui Ba Anh

    Thiếu úy

  • Thành viên
  • 562 Bài viết

$\boxed{\text{Giải bài 14.}}$ Gọi $D,E,F$ là tiếp điểm của $(I)$ với $BC,AC,AB$, giả sử $B_1,C_2$ thuộc $BC$,$B_2,A_1$ thuộc $AB$, $A_2,C_1$ thuộc $AC$, gọi các đường tròn nhỏ là $(u),(v),(w)$ thứ tự ứng với đỉnh $A,B,C$

Ta sẽ chứng minh $XD,YE,ZF$ đồng quy tại $P$

Thật vậy dễ thấy các tam giác $B_1XC_2,B_2A_1Z,YA_2C_1$ nội tiếp. (1)

Mà $\dfrac{sin(A_1ZF)}{sin(FZB_1)}=\dfrac{A_1F.ZB_2}{B_2F.ZA_1}$, thiết lập các đẳng thức tương tự và chú ý (1), áp dụng Ceva dạng sin ta có đpcm

Mặt khác tam giác $DEF$ đồng dạng $XYZ$ và $XD,YE,ZF$ đồng quy tại $P$ nên tồn tại phép vị tự tâm $P$ biến $DEF$ thành $XYZ$, do đó biến $I$ thành $M$ nên $P,I,M$ thẳng hàng.

Giả sử tiếp tuyến chung trong của $(v)$ và $(I)$ cắt $BD,XD,FZ$ tại $K,L,R$ và tiếp tuyến chung trong của $(w)$ và $(I)$ cắt $CD,XD,EY$ tại $H,L',Q$

Do $B_1K=KD$ và $DH=C_2H$ nên $DL=LX$ và $DL'=L'X$ theo talet hay các tiếp tuyến này cắt nhau tại một điểm trên $DX$

Chứng minh tương tự cho các tiếp tuyến chung còn lại

Khi đó tam giác $LQR$ đồng dạng $XYZ$ và $XL,YQ,CR$ đồng quy tại $P$ nên tồn tại phép vị tự biến $LQR$ thành $XYZ$ và do đó biến $I$ thành $N$ hay $P,I,N$ thẳng hàng

Vậy ta có đpcm.

$$\begin{array}{| l | l |} \hline Ngockhanh99k48 & 1\\ \hline IHateMath & 1\\ \hline fatcat12345 & 2\\ \hline dogsteven & 3\\ \hline baopbc & 4\\ \hline QuangDuong12011998 & 1\\ \hline xuantrandong & 1\\ \hline mrjackass & 1\\ \hline vietnaminmyheart & 1\\ \hline BuiBaAnh & 1\\ \hline\end{array}$$


Bài viết đã được chỉnh sửa nội dung bởi baopbc: 28-05-2016 - 22:21

NgọaLong

#31
viet nam in my heart

viet nam in my heart

    Thượng sĩ

  • Điều hành viên OLYMPIC
  • 242 Bài viết

$\boxed{\text{Bài toán 14.}}$ Em đưa ra lời giải của em, tương tự như cách của anh Bui Ba Anh

Các tiếp tuyến chung trong của các đường tròn $(O_a),(O_b),(O_c)$ với $(I)$ lần lượt là $d_1,d_2,d_3$ và chúng cắt nhau tạo thành tam giác $RST$. Theo tính chất của tiếp tuyến chung trong thì ta dễ dàng suy ra $R,S,T$ lần lượt là trung điểm của $XD,YE,ZF$

Mà $EF \parallel ST, DE \parallel RS, DF \parallel RT$ nên tồn tại một phép vị tự biến tam giác $DEF$ thành tam giác $RST$ hay $SE,RD,FT$ đồng quy tại $P$

Ta có: $I$ là tâm nội tiếp tam giác $RST$

Gọi $O_1$ là tâm ngoại tiếp tam giác $RST$. $I_1$ là tâm nội tiếp tam giác $DEF$

Xét phép vị tự biến tam giác $DEF$ thành tam giác $RST$: Khi đó $I_1,P,I$ thẳng hàng và $O_1,I,P$ thẳng hàng

Do đó $I_1,O_1,P,I$ thẳng hàng

Xét tiếp phép vị tự biến tam giác $DEF$ thành tam giác $RST$ và phép vị tự biến tam giác $RST$ thành tam giác $XYZ$

Khi đó: $P,I_1,I,N$ thẳng hàng (tâm nội tiếp của ba tam giác )

            $P,I,O_1,M$ thẳng hàng (tâm ngoại tiếp của ba tam giác )

Kết hợp với $I_1,O_1,P,I$ thẳng hàng thì ta dễ dàng suy ra các điểm $M,N,I,O_1,I_1,P$ thẳng hàng (điều phải chứng minh)

anh-dep-du-lich-vinh-yen-vinh-phuc-5-1024x707jpg_Page1.jpg

         


Bài viết đã được chỉnh sửa nội dung bởi viet nam in my heart: 29-05-2016 - 11:58

"Nếu bạn hỏi một người giỏi trượt băng làm sao để thành công, anh ta sẽ nói với bạn: ngã, đứng dậy là thành công." Isaac Newton

VMF's Marathon Hình học Olympic


#32
Bui Ba Anh

Bui Ba Anh

    Thiếu úy

  • Thành viên
  • 562 Bài viết

Mình xin lỗi vì quên đăng bài đề nghị

$\boxed{\text{Bài toán 15.}}$  Đường tròn $W_1$ và $W_2$ giao nhau tại $P,K$. $XY$ là tiếp tuyến chung ngoài gần $P$ hơn của $W_1,W_2$ với $X$ thuộc $W_1$ và $Y$ thuộc $W_2$. $XP$ cắt $W_2$ tại điểm thứ hai $C$ và $YP$ cắt $W_1$ tại điểm thứ hai $B$. Gọi $A$ là giao của $BX,CY$. Chứng minh rằng nếu $Q$ là giao điểm còn lại của $(ABC)$ và $(AXY)$ thì góc $QXA$ bằng góc $QKP$

Nguồn: sưu tầm

 

 


NgọaLong

#33
baopbc

baopbc

    Himura Kenshin

  • Thành viên nổi bật 2016
  • 410 Bài viết

$\boxed{\text{Lời giải bài 15}}$ Bài này dùng biến đổi góc là được! :)

Gọi $S$ là giao của $\odot (ABC)$ với $\odot (W_2)$.

Post 170.png

Ta có: $\measuredangle XYS=180^\circ-\measuredangle YCS=\measuredangle AQS=\measuredangle ABS\Longrightarrow X,Y,S,B$ đồng viên.

$\measuredangle XQS=\measuredangle AQS-\measuredangle AQX=\measuredangle XYS-\measuredangle AYX=180^\circ-\measuredangle YKS-\measuredangle AYX=180^\circ-\measuredangle YKS-\measuredangle YKX=180^\circ-\measuredangle XKS$

$\Longrightarrow Q,X,K,S$ đồng viên.

Giả sử $SY$ cắt $AB$ tại $T. \measuredangle TXK=180^\circ-\measuredangle BXK=180^\circ-\measuredangle BPK=180^\circ-\measuredangle TSK$

$\Longrightarrow T,X,K,S$ đồng viên $\Longrightarrow T,Q,K,X$ đồng viên $\Longrightarrow \measuredangle AXQ=\measuredangle TXQ$.

Mặt khác do $X,Y,S,B$ đồng viên nên $\overline{P,K,T}\Longrightarrow \measuredangle AXQ=\measuredangle QKP.\blacksquare$

Bài toán đề nghị.

$\boxed{\text{Bài toán 16}}$. Cho tam giác $ABC.\odot (K)$ bất kì qua $B,C$ cắt $CA,AB$ lần lượt tại $E,F.BE,CF$ cắt nhau tại $H$. Kẻ $HL,BM,CN$ vuông góc với $AK(M,N,P$ thuộc $AK$). Gọi $G$ là hình chiếu của $L$ lên $BC$. Đường thẳng qua $A$ lần lượt song song với $BE,CF$ lần lượt cắt trung trực $AM,AN$ tại $Q,R$. Chứng minh rằng trục đẳng phương của $\odot (Q;QA),\odot (R;RA)$ đi qua $G$.

Post 174.png

Nguồn.


Bài viết đã được chỉnh sửa nội dung bởi halloffame: 30-12-2017 - 13:07


#34
viet nam in my heart

viet nam in my heart

    Thượng sĩ

  • Điều hành viên OLYMPIC
  • 242 Bài viết

$\boxed{\text{Lời giải bài 16}}$: Lời giải của mình không được hay cho lắm

Dễ thấy chỉ cần chứng minh $QR \perp AG$ là xong

Gọi $X,Y$ lần lượt là điểm đối xứng của $A$ qua $Q,R$. Suy ra $B,M,X$ thẳng hàng và $C,N,Y$ thẳng hàng

Khi đó $QR$ là đường trung bình của tam giác $AXY$ nên $QR \parallel XY$ suy ra chỉ cần chứng minh $XY \perp AG$ 

Theo một bổ đề quen thuộc khi chứng minh định lý $Brocard$ thì ta có: $HL,EF,BC$ đồng quy tại $U$

                                                                                                                  $E,F,H,L$ đồng viên

                                                                                                                  $B,C,H,L$ đồng viên

Đường thẳng qua $A$ vuông góc với $AK$ cắt $BE,CF$ tại $T,F$

Do $HL,EF,BC$ đồng quy tại $U$ nên ta có chùm điều hòa cơ bản $H(STAL)=-1$ mà $ST \parallel HL$ nên $AS=AT$

Ta có:$ATBX,ASCY$ là hình bình hành nên suy ra $BXCY$ là hình bình hành suy ra $BY,CX$ cắt nhau tại trung điểm $Z$ của mỗi đường

Ta có: $\overrightarrow{XY}=2\overrightarrow{XB}+\overrightarrow{BC}=\overrightarrow{ST}+\overrightarrow{BC}$ và $\overrightarrow{AG}=\overrightarrow{AL}+\overrightarrow{LG}$

Cần chứng minh $\overrightarrow{XY}\overrightarrow{LG}=0 \Leftrightarrow \left(\overrightarrow{ST}+\overrightarrow{BC}\right)\left(\overrightarrow{AL}+\overrightarrow{LG}\right)=0$ 

Để ý: $AL \perp ST,BC \perp LG$ nên sau khi khai triển chỉ cần chứng minh: $\overrightarrow{ST}\overrightarrow{LG}+\overrightarrow{AL}\overrightarrow{BC}=0$

Giả sử $GL$ cắt $ST$ tại $I$, $AL$ cắt $BC$ tại $J$. Suy ra $AIJG$ nội tiếp. Từ đó: $\widehat{AIG}=\widehat{AJB}$ 

Do đó ta có thể bỏ ký hiệu vectơ hay chỉ cần chứng minh: $AL.BC=LG.ST \Leftrightarrow \dfrac{AL}{ST}=\dfrac{LG}{BC}$

Đến đây mình chưa tìm được cách đẹp chỉ đành biến đổi tỷ số lượng giác:

Ta có: $\dfrac{ST}{AL}=\dfrac{ST}{SH}.\dfrac{SH}{AH}.\dfrac{AH}{AL}=\dfrac{sin\widehat{SHT}}{sin\widehat{STH}}\dfrac{sin\widehat{SAH}}{sin\widehat{HSA}}\dfrac{1}{sin\widehat{AHL}}=\dfrac{sin\widehat{BHC}}{sin\widehat{LCB}.sin\widehat{LBC}}$

Lại có: $\dfrac{LG}{BC}=\dfrac{LG}{LB}\dfrac{LB}{BC}=sin \widehat{LBC}.\dfrac{\sin\widehat{LCB}}{sin\widehat{BLC}}=\dfrac{AL}{ST}$

Suy ra điều phải chứng minh.

anh-dep-du-lich-vinh-yen-vinh-phuc-5-1024x707jpg_Page1.jpg

Bài toán đề nghị.

$\boxed{\text{Bài toán 17}}$ Sưu tầm Cho tam giác $ABC$ nhọn. Đường cao $BE,CF$ và trực tâm $H$. Gọi $M$ là trung điểm $BC$. Giả sử $(MEF)$ cắt $(BHC)$ tại $P,Q$.( $P$ thuộc cung $BH$, $Q$ thuộc cung $CH$). Gọi $M$ là trung điểm $BC$. Chứng minh rằng $MP,QH,EF$ đồng quy

anh-dep-du-lich-vinh-yen-vinh-phuc-5-1024x707jpg_Page1.jpg


Bài viết đã được chỉnh sửa nội dung bởi halloffame: 30-12-2017 - 13:07

"Nếu bạn hỏi một người giỏi trượt băng làm sao để thành công, anh ta sẽ nói với bạn: ngã, đứng dậy là thành công." Isaac Newton

VMF's Marathon Hình học Olympic


#35
xuantrandong

xuantrandong

    Hạ sĩ

  • Thành viên
  • 62 Bài viết

$\boxed{\text{Lời giải bài toán 17}}$

Gọi $X$ và $Y$ là giao điểm của $EF$ và đường tròn ngoại tiếp tam giác $ABC$

xét phép nghịch đảo cực $A$ phương tích $AX^{2}=AY^{2}=AF.AB=AE.AC$ ta có đường tròn $(A;AX)$ trực giao với đường tròn đường kính $BC$ hay$P_{M,(A;AX)}=MB^{2}=MC^{2}$ 

Gọi $K$ là giao điểm $EF$ và $BC$

Gọi $U$ và $V$ là giao điểm của $MX,MY$ và  $(A;AX)$

Do $(K,D,B,C)=-1$ nên theo $Maclaurin$ ta có $KD.KM=KB.KC=KX.KY$ suy ra $XYMD$ nội tiếp $\Rightarrow \angle DXU=\angle DYV$ 

Do $(K,D,B,C)=-1$ nên theo $Newton$ ta có $MX.MU=MB^{2}=MD.MK$ suy ra $XUDK$ nội tiếp

tương tự $YVDK$ nội tiếp

ta có $\angle DKU=\angle DXU=\angle DYV=\angle DKV$ nên $K,U,V$ thẳng hàng

mà $MF^{2}=MD.MK$ nên $\angle DFM=\angle DKF=\angle DUM$ nên $DMFU$ nội tiếp tương tự $DMEV$ nội tiếp

$\Rightarrow $ đường thằng chứa $K,U,V$ chính là trục đẳng phương của đường tròn $Euler$ và   $(A;AX)$

ta có tâm đường tròn $Euler$, tâm $A'$ của $(BHC)$ và $A$ thằng hàng  nên $AA'$ vuông góc  $UV$

mà $KX.KY=KB.KC= KU.KV$ và $K$ thuộc đương thẳng $UV$ nên $(A;AX)$, đường tròn $Euler$ và đường tròn $(BHC)$ có 2 điểm chung là $U,V$

vậy  $U\equiv P,V\equiv Q$

Áp dụng định lý $Brocard$ cho tứ giác $BECF$ nội tiếp ta có $H$ là trực tâm của tam giác $AKM$, mà $A$ là tâm  $(A;AX)$  nên theo định lý $Brocard$ cho tứ giác nội tiếp $XYPQ$ thì $H$ là giao điểm $XQ,YP\Rightarrow XQ$ đi qua $H$

vậy $MP,QH,EF$ đồng quy tại $X$ ta có đpcm.

Untitled.jpg


Bài viết đã được chỉnh sửa nội dung bởi halloffame: 30-12-2017 - 13:08


#36
quanghung86

quanghung86

    Thiếu úy

  • Điều hành viên
  • 632 Bài viết

$\boxed{\text{Lời giải bài 16}}$: Lời giải của mình không được hay cho lắm

Dễ thấy chỉ cần chứng minh $QR \perp AG$ là xong

Gọi $X,Y$ lần lượt là điểm đối xứng của $A$ qua $Q,R$. Suy ra $B,M,X$ thẳng hàng và $C,N,Y$ thẳng hàng

Khi đó $QR$ là đường trung bình của tam giác $AXY$ nên $QR \parallel XY$ suy ra chỉ cần chứng minh $XY \perp AG$ 

Theo một bổ đề quen thuộc khi chứng minh định lý $Brocard$ thì ta có: $HL,EF,BC$ đồng quy tại $U$

                                                                                                                  $E,F,H,L$ đồng viên

                                                                                                                  $B,C,H,L$ đồng viên

Đường thẳng qua $A$ vuông góc với $AK$ cắt $BE,CF$ tại $T,F$

Do $HL,EF,BC$ đồng quy tại $U$ nên ta có chùm điều hòa cơ bản $H(STAL)=-1$ mà $ST \parallel HL$ nên $AS=AT$

Ta có:$ATBX,ASCY$ là hình bình hành nên suy ra $BXCY$ là hình bình hành suy ra $BY,CX$ cắt nhau tại trung điểm $Z$ của mỗi đường

 

Lời giải của viet nam in my heart rất hay nhưng để mình chỉnh lại một chút đoạn sau cho đẹp, không cần tích vô hướng.

 

Đến đoạn "$BY,CX$ cắt nhau tại trung điểm $Z$ của mỗi đường". (Chú ý $A$ là trung điểm $ST$ có thể dễ thấy qua bài toán con bướm)

 

 

Ta cần chứng minh hai tam giác $BZX$ và $LGA$ đồng dạng là xong vì đã có $LG\perp BZ,LA\perp BX$. Vậy ta cần cm $\frac{LG}{LA}=\frac{BZ}{BX}$ hay $\frac{LG}{LA}=\frac{BC}{ST}.$

Kẻ đường cao $HJ$ của $HST$ thì $LA=HJ$. Mặt khác từ $\triangle HST\sim\triangle LBC$ nên $\frac{AL}{ST}=\frac{HJ}{ST}=\frac{LG}{BC}$. Ta có đpcm.
 


Bài viết đã được chỉnh sửa nội dung bởi halloffame: 30-12-2017 - 13:21


#37
xuantrandong

xuantrandong

    Hạ sĩ

  • Thành viên
  • 62 Bài viết

Mình xin đề xuất $\boxed{\text{Bài toán 18}}$

(Sưu tầm) Cho$\Delta ABC$ nhọn. $M$ là một điểm di động trên cạnh $AB$ và $N$ là trung điểm $AC$. Gọi $P,Q$ là hình chiếu của $A$ trên $MC,MN$. Chứng minh rằng khi $M$ di động trên cạnh $AB$ thì tâm đường tròn ngoại tiếp $\Delta PQN$ nằm trên một đường thẳng cố định.


Bài viết đã được chỉnh sửa nội dung bởi xuantrandong: 31-05-2016 - 13:36


#38
Ngockhanh99k48

Ngockhanh99k48

    Trung sĩ

  • Thành viên
  • 127 Bài viết

$\boxed{\text{Lời giải bài toán 18:}}$
Đường thẳng qua $C$ song song $AB$ cắt đường thẳng qua $A$ vuông góc $AB$ tại $H$. Ta có $\triangle AHC$ vuông tại $H$ nên $\triangle NHC$ cân tại $N$. Do đó $\widehat{NHP}=\widehat{NHC}-\widehat{PHC}=\widehat{NCH}-\widehat{PAC}$(do $P, A, H, C$ đồng viên) $= \widehat{BAC}-\widehat {PAC}=\widehat{BAP}=\widehat{MQP}$. Do đó $(PQN)$ đi qua điểm $H$ cố định hay tâm $(PQN)$ thuộc trung trực $NH$ cố định. Ta có đpcm.

Untitled.jpg
P/s: mãi mới tranh bài để được giải, các bạn giải nhanh quá :(. Mình xin đề xuất bài toán 19 sau, mình sưu tầm được từ thầy Nguyễn Minh Hà
$\boxed{\text{Bài toán 19: }}$ Cho tam giác $\triangle ABC$ nội tiếp $(O)$, ngoại tiếp $(I)$. Các điểm $A_b, A_c$ thứ tự thuộc các cạnh $AB, AC$ sao cho $A_bA_c$ song song $BC$ và $A_bA_c$ tiếp xúc $(I)$. Định nghĩa tương tự với các điểm $B_c, B_a, C_a, C_b$. Gọi $I_1, I_2, I_3$ là tâm nội tiếp $\triangle AA_bA_c$, $\triangle BB_cB_a$, $\triangle CC_aC_b$. Chứng minh rằng $OI$ đi qua điểm Lemoine của $\triangle I_1I_2I_3$.

$$\begin{array}{| l | l |} \hline Ngockhanh99k48 & 2\\ \hline IHateMath & 1\\ \hline fatcat12345 & 2\\ \hline dogsteven & 3\\ \hline baopbc & 5\\ \hline QuangDuong12011998 & 1\\ \hline xuantrandong & 2\\ \hline mrjackass & 1\\ \hline vietnaminmyheart & 2\\ \hline BuiBaAnh & 1\\ \hline\end{array}$$


Bài viết đã được chỉnh sửa nội dung bởi baopbc: 31-05-2016 - 18:21
Bảng điểm


#39
QuangDuong12011998

QuangDuong12011998

    Hạ sĩ

  • Thành viên
  • 50 Bài viết
Lời giải sử dụng vector cho bài 19.
Đầu tiên là do thói quen, cho nên mình thay $I_1$, $I_2$, $I_3$ bởi $A'$, $B'$, $C'$. Trong chứng minh này, kí hiệu $a$, $b$, $c$ là độ dài $BC$, $CA$, $AB$ và $p$ là nửa chu vi $\triangle ABC$.
$\triangle ABC$ và $\triangle AA_bA_c$ vị tự tâm $A$, bây giờ ta cần tính tỉ số vị tự. Hai tam giác đồng dạng thì tỉ số hai đường cao bằng tỉ số đồng dạng. Đường cao ứng với $A$ của $\triangle ABC$ có độ dài $h_a$, đường cao ứng với $A$ của $\triangle AA_bA_c$ có độ dài $h_a-2r$.
\[\dfrac{h_a-2r}{h_a}=1-\dfrac{2r}{h_a}=1-\dfrac{2\dfrac{S}{p}}{\dfrac{2S}{a}}=\dfrac{p-a}{p}\]
Từ đó mà $\dfrac{\overline{AA'}}{\overline{AI}}=\dfrac{p-a}{p}$ từ điều này ta dễ dàng thu được $\overrightarrow{IA'}=\dfrac{a\overrightarrow{IA}}{p}$. Tương tự ta có
\[\overrightarrow{IB'}=\dfrac{b\overrightarrow{IB}}{p}\qquad \overrightarrow{IC'}=\dfrac{c\overrightarrow{IC}}{p}\]
Ta sẽ xem độ dài ba cạnh $\triangle A'B'C'$ tỉ lệ với ba giá trị nào(hi vọng là đơn giản). Mục đích của việc này mình sẽ giải thích bên dưới.
\[\overrightarrow{B'C'}=\dfrac{c}{p}\overrightarrow{IC}-\dfrac{b}{p}\overrightarrow{IB}\]
\[\Rightarrow B'C'^2=\dfrac{c^2IC^2+b^2IB^2-2bc\overrightarrow{IB}\cdot\overrightarrow{IC}}{p^2}\]
\[p^2B'C'^2=\dfrac{ab^2c(p-b)}{p}+\dfrac{abc^2(p-c)}{p}-bc(IB^2+IC^2-a^2)\]
\[p^2B'C'^2=\dfrac{abc(pb+pc-b^2-c^2)}{p}+a^2bc-bc(\dfrac{ca(p-b)+ab(p-c)}{p})\]
\[p^2B'C'^2=\dfrac{-abc(b-c)^2}{p}+a^2bc\]
Tương tự ta tính  $B'C'$, $C'A'$, $A'B'$ thì $B'C'^2$, $C'A'^2$, $A'B'^2$ tỉ lệ với $pa-(b-c)^2$,$ pb-(c-a)^2$, $pc-(a-b)^2$
Điểm $K$ là điểm Lemoine của $\triangle A'B'C'$ nên bộ số tâm tỉ cự của $K$ với $\triangle A'B'C'$ chính là $pa-(b-c)^2$,$ pb-(c-a)^2$, $pc-(a-b)^2$.
Do đó
\[(pa+pb+pc-(b-c)^2-(c-a)^2-(a-b)^2)\overrightarrow{OK}=\sum(pa-(b-c)^2)\overrightarrow{OA'}\]
Điểm $K$ là điểm Lemoine của $\triangle A'B'C'$ nên bộ số tâm tỉ cự của $K$ với $\triangle A'B'C'$ chính là $pa-(b-c)^2$,$ pb-(c-a)^2$, $pc-(a-b)^2$.
Do đó
\[(pa+pb+pc-(b-c)^2-(c-a)^2-(a-b)^2)\overrightarrow{OK}=\sum(pa-(b-c)^2)\overrightarrow{OA'}\]
\[(pa+pb+pc-(b-c)^2-(c-a)^2-(a-b)^2)\overrightarrow{OK}=\sum(pa-(b-c)^2)\dfrac{(p-a)\overrightarrow{OI}+a\overrightarrow{OA}}{p}\]
\[(pa+pb+pc-(b-c)^2-(c-a)^2-(a-b)^2)\overrightarrow{OK}=\mathcal{P}\overrightarrow{OI}+\sum(pa^2-a(b-c)^2)\overrightarrow{OA}\]
\[(pa+pb+pc-(b-c)^2-(c-a)^2-(a-b)^2)\overrightarrow{OK}=\mathcal{P}\overrightarrow{OI}+\mathcal{Q}\overrightarrow{OT}\]
Trong đó $\mathcal{P}$ là một biểu thức. Bây giờ ta quan tâm tới điểm $T$ mà $\sum (pa^2-a(b-c)^2)\overrightarrow{TA}=\overrightarrow{0}$. Chỉ cần chỉ ra $O$, $I$, $T$ thẳng hàng là được. Bộ số tâm tỉ cự của $I(a:b:c)$ và của $O(a^2(b^2+c^2-a^2):\ldots:\ldots)$. Theo tài liệu tổng kêt sau https://blogcuaquang...oa-o-ti-cu.html thì $O,I,T$ thẳng hàng khi và chỉ khi
\[\sum(pa^2-a(b-c)^2)(bc^2(a^2+b^2-c^2)-b^2c(c^2+a^2-b^2))=0\]
Biến đổi tương đương, phân tích nhân tử (cái này thì mình xài wolframalpha) thì tương đương
\[abcp(b-c)(c-a)(a-b)(a+b+c-2p)=0\]
rõ ràng đẳng thức đúng vì $a+b+c=2p$
nên $O,I,T$ thẳng hàng, mà theo trên các vector $\overrightarrow{OK}$ biểu diễn được theo $\overrightarrow{OT}$, $\overrightarrow{OI}$ nên kéo theo $O$, $I$, $K$ thẳng hàng.
OI.png
$\boxed{\text{Bài toán 20}}$(Yahoo) Cho 4 điểm $A,B,C,D$. Chứng minh đường tròn Miquel của 3 tứ giác toàn phần $(AB,AC,DB,DC)$, $(BC,BA,DC,DA)$, $(CA,CB,DA,DB)$ đồng quy.
$$\begin{array}{| l | l |} \hline Ngockhanh99k48 & 2\\ \hline IHateMath & 1\\ \hline fatcat12345 & 2\\ \hline dogsteven & 3\\ \hline baopbc & 5\\ \hline QuangDuong12011998 & 2\\ \hline xuantrandong & 2\\ \hline mrjackass & 1\\ \hline vietnaminmyheart & 2\\ \hline BuiBaAnh & 1\\ \hline\end{array}$$

Bài viết đã được chỉnh sửa nội dung bởi QuangDuong12011998: 01-06-2016 - 14:32


#40
quanghung86

quanghung86

    Thiếu úy

  • Điều hành viên
  • 632 Bài viết

Cấu hình của bài thầy Hà trong bài đề nghị của Khánh là cấu hình rất thú vị, trên đó khai thác được khá nhiều điểm đặc biệt các tam giác $AA_bA_c,BB_cB_a,CC_aC_b$ mà những tâm đặc biệt của tam giác tạo bởi các tâm trên nằm trên đường thẳng $OI$ của $ABC$.

 

Mình ví dụ một bài như sau

 

Giữ nguyên đề của Khánh. Gọi $O_a,O_b,O_c$ là tâm ngoại tiếp tam giác $AA_bA_c,BB_cB_a,CC_aC_b$. Chứng minh rằng trọng tâm tam giác $O_aO_bO_c$ nằm trên đường thẳng $OI$ của $ABC$.


Bài viết đã được chỉnh sửa nội dung bởi quanghung86: 01-06-2016 - 17:01
Nhầm tâm ngoại tiếp và trọng tâm.






Được gắn nhãn với một hoặc nhiều trong số những từ khóa sau: hình học

3 người đang xem chủ đề

0 thành viên, 2 khách, 0 thành viên ẩn danh


    Google (1)